K
Khách

Hãy nhập câu hỏi của bạn vào đây, nếu là tài khoản VIP, bạn sẽ được ưu tiên trả lời.

10 tháng 1 2017

\(P=\frac{1}{a^3\left(b+c\right)}+\frac{1}{b^3\left(a+c\right)}+\frac{1}{c^3\left(a+b\right)}\)

\(=\frac{bc}{a^2\left(b+c\right)}+\frac{ac}{b^2\left(a+c\right)}+\frac{ab}{c^2\left(a+b\right)}\left(abc=1\right)\)

\(=\frac{1}{a^2\left(\frac{1}{c}+\frac{1}{b}\right)}+\frac{1}{b^2\left(\frac{1}{c}+\frac{1}{a}\right)}+\frac{1}{c^2\left(\frac{1}{b}+\frac{1}{a}\right)}\)

\(=\frac{\frac{1}{a^2}}{\frac{1}{c}+\frac{1}{b}}+\frac{\frac{1}{b^2}}{\frac{1}{c}+\frac{1}{a}}+\frac{\frac{1}{c^2}}{\frac{1}{b}+\frac{1}{a}}\)

Đặt \(\left\{\begin{matrix}\frac{1}{a}=x\\\frac{1}{b}=y\\\frac{1}{c}=z\end{matrix}\right.\) suy ra \(xyz=1\). Khi đó:

\(P=\frac{x^2}{y+z}+\frac{y^2}{z+x}+\frac{z^2}{x+y}\)

Áp dụng BĐT AM-GM ta có:

\(\left\{\begin{matrix}\frac{x^2}{y+z}+\frac{y+z}{4}\ge x\\\frac{y^2}{x+z}+\frac{x+z}{4}\ge y\\\frac{z^2}{x+y}+\frac{x+y}{4}\ge z\end{matrix}\right.\).Cộng theo vế ta có:

\(P+\frac{x+y+z}{2}\ge x+y+z\)

\(\Rightarrow P\ge\frac{x+y+z}{2}\ge\frac{3}{2}\left(x+y+z\ge3\sqrt[3]{xyz}=3\right)\)

12 tháng 1 2017

Đại số lớp 8

7 tháng 3 2020

Ồ sorry bạn nhiều, chỗ đấy bị lỗi kĩ thuật rồi, mình sửa lại nhé :

\(M\ge\frac{\left(\frac{1}{a}+\frac{1}{b}+\frac{1}{c}\right)^2}{2\left(ab+bc+ca\right)}=\frac{\left(ab+bc+ca\right)^2}{2\left(ab+bc+ca\right)}=\frac{ab+bc+ca}{2}\)

Lại có : \(\frac{ab+bc+ca}{2}\ge\frac{3\sqrt{a^3b^3c^3}}{2}=\frac{3}{2}\)

Do đó : \(M\ge\frac{3}{2}\)

Dấu "=" xảy ra \(\Leftrightarrow a=b=c=1\)

7 tháng 3 2020

Ta có : \(\frac{1}{a^3\left(b+c\right)}=\frac{\frac{1}{a^2}}{a\left(b+c\right)}=\frac{\left(\frac{1}{a}\right)^2}{a\left(b+c\right)}\)

Tương tự : \(\frac{1}{b^3\left(a+c\right)}=\frac{\left(\frac{1}{b}\right)^2}{b\left(a+c\right)}\) , \(\frac{1}{c^3\left(a+b\right)}=\frac{\left(\frac{1}{c}\right)^2}{c\left(a+b\right)}\)

Ta thấy : \(\left(a-b\right)^2+\left(b-c\right)^2+\left(c-a\right)^2\ge0\)

\(\Leftrightarrow a^2+b^2+c^2\ge ab+bc+ca\)

\(\Leftrightarrow\left(a+b+c\right)^2\ge3\left(ab+bc+ca\right)\)

Áp dụng BĐT Svacxo ta có :

\(M=\frac{1}{a^3\left(b+c\right)}+\frac{1}{b^2\left(a+c\right)}+\frac{1}{c^3\left(a+b\right)}\ge\frac{\left(\frac{1}{a}+\frac{1}{b}+\frac{1}{b}\right)^2}{2\left(ab+bc+ca\right)}=\frac{\left(a+b+c\right)^2}{2\left(ab+bc+ca\right)}\)   \(\ge\frac{3\left(ab+bc+ca\right)}{2\left(ab+bc+ca\right)}=\frac{3}{2}\)

Dấu "=" xảy ra \(\Leftrightarrow a=b=c=1\)

Vâỵ \(M_{min}=\frac{3}{2}\) tại \(a=b=c=1\)

15 tháng 5 2016

ta có : \(\frac{1}{a^3\left(b+c\right)}+\frac{1}{b^3\left(a+c\right)}+\frac{1}{c^3\left(a+b\right)}=\frac{\frac{1}{a^2}}{a\left(b+c\right)}+\frac{\frac{1}{b^2}}{b\cdot\left(a+c\right)}+\frac{\frac{1}{c^2}}{c\left(a+b\right)}\)(1)

dùng Svaxo ta có (1) <=>\(\frac{\left(\frac{1}{a^2}+\frac{1}{b^2}+\frac{1}{c^2}\right)^2}{a\left(b+c\right)+b\left(c+a\right)+c\left(a+b\right)}=\frac{ab+bc+ca}{2}>=\frac{3a^2b^2c^2}{2}=\frac{3}{2}\)(côsi )

15 tháng 5 2016

mik viết nhầm phải là (1) >=

26 tháng 2 2021

Theo bđt Cauchy - Schwart ta có:

\(\text{Σ}cyc\frac{c}{a^2\left(bc+1\right)}=\text{Σ}cyc\frac{\frac{1}{a^2}}{b+\frac{1}{c}}\ge\frac{\left(\frac{1}{a}+\frac{1}{b}+\frac{1}{c}\right)^2}{\frac{1}{a}+\frac{1}{b}+\frac{1}{c}+a+b+c}\)\(=\frac{\left(\frac{1}{a}+\frac{1}{b}+\frac{1}{c}\right)^2}{\frac{1}{a}+\frac{1}{b}+\frac{1}{c}+3}\)

\(=\frac{\left(ab+bc+ca\right)^2}{abc\left(ab+bc+ca\right)+3a^2b^2c^2}\)

Đặt \(ab+bc+ca=x;abc=y\).

Ta có: \(\frac{x^2}{xy+3y^2}\ge\frac{9}{x\left(1+y\right)}\Leftrightarrow x^3+x^3y\ge9xy+27y^2\)

\(\Leftrightarrow x\left(x^2-9y\right)+y\left(x^3-27y\right)\ge0\) ( luôn đúng )

Vậy BĐT đc CM. Dấu '=' xảy ra <=> a=b=c=1

26 tháng 2 2021

sai rồi nhé bạn 

11 tháng 4 2017

\(\frac{a^3}{\left(1+b\right)\left(1+c\right)}+\frac{b^3}{\left(1+c\right)\left(1+a\right)}+\frac{c^3}{\left(1+a\right)\left(1+b\right)}\)

Ta có:

\(\frac{a^3}{\left(1+b\right)\left(1+c\right)}+\frac{1+b}{8}+\frac{1+c}{8}\ge\frac{3a}{4}\)

\(\Leftrightarrow\frac{a^3}{\left(1+b\right)\left(1+c\right)}\ge\frac{6a-b-c-2}{8}\)

Tương tự ta có: \(\hept{\begin{cases}\frac{b^3}{\left(1+c\right)\left(1+a\right)}\ge\frac{6b-c-a-2}{8}\\\frac{c^3}{\left(1+a\right)\left(1+b\right)}\ge\frac{6c-a-b-2}{8}\end{cases}}\)

Cộng vế theo vế ta được

\(\frac{a^3}{\left(1+b\right)\left(1+c\right)}+\frac{b^3}{\left(1+c\right)\left(1+a\right)}+\frac{c^3}{\left(1+a\right)\left(1+b\right)}\ge\frac{6a-b-c-2}{8}+\frac{6b-c-a-2}{8}+\frac{6c-a-b-2}{8}\)

\(=\frac{a+b+c}{2}-\frac{3}{4}\ge\frac{3}{2}.\sqrt[3]{abc}-\frac{3}{4}=\frac{3}{2}-\frac{3}{4}=\frac{3}{4}\)

10 tháng 4 2017

Mai mình làm cho

14 tháng 3 2016

Sử dụng bất đẳng thức  \(\frac{a}{x}+\frac{b}{y}+\frac{c}{z}\ge\frac{\left(a+b+c\right)^2}{x+y+z}\)  với ba số  \(a,b,c\)  và ba số  dương \(x,y,z\)  bất kỳ với chú ý rằng  \(a^2b^2c^2=1\), ta có:

 \(\frac{1}{a^3\left(b+c\right)}+\frac{1}{b^3\left(c+a\right)}+\frac{1}{c^3\left(a+b\right)}=\frac{b^2c^2}{a\left(b+c\right)}+\frac{c^2a^2}{b\left(c+a\right)}+\frac{a^2b^2}{c\left(a+b\right)}\ge\frac{\left(ab+bc+ca\right)^2}{2\left(ab+bc+ca\right)}=\frac{ab+bc+ca}{2}\)  \(\left(1\right)\)

Đặt  \(x=ab;\)  \(y=bc;\)  và  \(z=ca\)  thì  \(xyz=1\)  \(\left(2\right)\) với  \(x;\)\(y;\) và  \(z\)  \(>0\)  

Khi đó áp dụng BĐT Cauchy cho bộ ba số nguyên dương \(x;\)\(y;\) và  \(z\), ta được:

\(x+y+z\ge3\sqrt[3]{xyz}\)  \(\Leftrightarrow\)  \(x+y+z\ge3\)  (do  \(\left(2\right)\)), tức  \(ab+bc+ca\ge3\)  \(\left(3\right)\)

Từ  \(\left(1\right);\)  \(\left(3\right)\) ta suy ra   \(\frac{1}{a^3\left(b+c\right)}+\frac{1}{b^3\left(c+a\right)}+\frac{1}{c^3\left(a+b\right)}\ge\frac{3}{2}\)  \(\left(đpcm\right)\)

Dấu  \("="\)  xảy ra  khi và chỉ khi  \(a=b=c=1\)

14 tháng 3 2016

thông điệp nhỏ :

hãy tích nếu như ko muốn tích 

ai tích mình tích lại nh nha 

19 tháng 11 2019

Áp dụng BĐT AM - GM:

\(\frac{3}{2}\ge a+b+c\ge3\sqrt[3]{abc}\) \(\Rightarrow abc\le\frac{1}{8}\)

\(1+1+1+\frac{1}{2a}+\frac{1}{2a}+\frac{1}{2b}+\frac{1}{2b}\ge7\sqrt[7]{\frac{1}{16a^2b^2}}\)

\(\Leftrightarrow3+\frac{1}{a}+\frac{1}{b}\ge7\sqrt[7]{\frac{1}{16a^2b^2}}\)

Tương tự ta CM được:

\(3+\frac{1}{b}+\frac{1}{c}\ge7\sqrt[7]{\frac{1}{16b^2c^2}}\)

\(3+\frac{1}{c}+\frac{1}{a}\ge\ge7\sqrt[7]{\frac{1}{16c^2a^2}}\)

Nhân vế theo vế 3 bất đẳng thức trên:

\(S\ge343\sqrt[7]{\frac{1}{4096a^4b^4c^4}}\ge343\sqrt[7]{\frac{1}{4096.\frac{1}{8^4}}}=343\)

\(\Rightarrow Min_S=343\Leftrightarrow a=b=c=\frac{1}{2}\)

19 tháng 11 2019

@Nguyễn Việt Lâm